Critical Reasoning

This topic has expert replies
Moderator
Posts: 772
Joined: Wed Aug 30, 2017 6:29 pm
Followed by:6 members

Critical Reasoning

by BTGmoderatorRO » Tue Feb 06, 2018 2:46 pm
To decrease the number of crimes in city Y, the city's Police Commissioner proposed taking some police officers from low-crime districts of the city and moving them to high-crime districts of the city. His proposal is based on city Y crime data that show that the number of crimes in any district of the city decreases when additional police officers are moved into that district.

The Police Commissioner's proposal depends on which of the following assumptions?


(A) City X experienced a drastic reduction in crime after implementing a proposal similar to that proposed by the Police Commissioner of city Y.

(B) The severity of crimes committed in any district of the city decreases when additional police officers are moved into that district.

(C) The number of crimes committed in all high-crime districts of city Y is more than triple the number of crimes committed in all low-crime districts of city Y.

(D) There are more low-crime districts than high-crime districts in city Y.

(E) Districts of the city from which police officers are removed do not experience significant crime increases shortly after the removal of those officers.

OA is E
Please what makes option B wrong in this question. I need a concrete explanation from an Expert. Thanks a lot

User avatar
Legendary Member
Posts: 2663
Joined: Wed Jan 14, 2015 8:25 am
Location: Boston, MA
Thanked: 1153 times
Followed by:128 members
GMAT Score:770

by DavidG@VeritasPrep » Wed Feb 07, 2018 5:52 am
Roland2rule wrote:To decrease the number of crimes in city Y, the city's Police Commissioner proposed taking some police officers from low-crime districts of the city and moving them to high-crime districts of the city. His proposal is based on city Y crime data that show that the number of crimes in any district of the city decreases when additional police officers are moved into that district.

The Police Commissioner's proposal depends on which of the following assumptions?


(A) City X experienced a drastic reduction in crime after implementing a proposal similar to that proposed by the Police Commissioner of city Y.

(B) The severity of crimes committed in any district of the city decreases when additional police officers are moved into that district.

(C) The number of crimes committed in all high-crime districts of city Y is more than triple the number of crimes committed in all low-crime districts of city Y.

(D) There are more low-crime districts than high-crime districts in city Y.

(E) Districts of the city from which police officers are removed do not experience significant crime increases shortly after the removal of those officers.

OA is E
Please what makes option B wrong in this question. I need a concrete explanation from an Expert. Thanks a lot
Try negation - the correct answer, when negated, will undermine the argument.

E negated: Districts of the city from which police officers are removed DO experience significant crime increases shortly after the removal of those officers.

Well, if districts experience a crime spike once police officers are removed, then the plan to shift officers from low-crime areas to high-crime areas isn't like to reduce overall crime. Rather, it will cause crime to increase in areas that the police are moved from and cause crime to decrease in areas that the police are moved to. Because E, when negated, undermines the argument, we know it's the correct answer.
Veritas Prep | GMAT Instructor

Veritas Prep Reviews
Save $100 off any live Veritas Prep GMAT Course